Использование функционалов в формализме Мартина Сиггиа Роуза

В настоящее время я изучаю «Критическую динамику - подход теории поля к равновесному и неравновесному масштабирующему поведению» и столкнулся с проблемой, которую не могу решить.

Если вы знакомы с функциональными производными и/или формализмом Мартина-Сиггиа-Роуза, пожалуйста, пропустите довольно длинный абзац, чтобы внести некоторый контекст. Я надеюсь, что обозначения стандартны (я новичок в этой области).

Контекст

Если у вас есть доступ к книге (к сожалению, общедоступной версии нет), начиная с главы 4, для динамики Ланжевена настроен формализм Мартина-Сиггиа-Роуза, в данном случае пример:

Обобщая гамильтонианы Ландау–Гинзбурга–Вильсона для моделей Изинга и Гейзенберга[...] на n-компонентные изотропные системы, испытывающие фазовый переход второго рода, мы рассматриваем

ЧАС [ С ] "=" г г Икс α ( р 2 [ С α ( Икс ) ] 2 + 1 2 [ С α ( Икс ) ] 2 + ты 4 ! β [ С α ( Икс ) ] 2 [ С β ( Икс ) ] 2 час α ( Икс ) С α ( Икс ) )
С С n-компонентный параметр порядка и h сопряженное внешнее поле.

Ассоциированная эволюция представляет собой уравнение Ланжевена, определяемое как:

С α ( Икс , т ) т "=" Д ( я ) а дельта ЧАС [ С ] дельта С α ( Икс , т ) + ζ α ( Икс , т )

Если колебания учитываются через ζ , диагональный белый шум с нулевым средним значением ζ α ( Икс , т ) ζ β ( Икс , т ) "=" 2 л дельта ( Икс Икс ) дельта ( т т ) дельта α β , и л "=" Д ( я ) а .

Отсюда выводится производящий функционал для корреляционных функций,

Z [ Дж ~ , Дж ] "=" е Икс п г г Икс г т α Дж α ~ ( Икс , т ) С α ~ ( Икс , т ) + Дж α ( Икс , т ) С α ( Икс , т )

The Дж - это исходные токи, а тильда - вспомогательные поля MSR / исходные токи.

С помощью функциональных производных по токам, взятых при нулевых токах, получают корреляционные функции.

Логарифмирование Z дает производящий функционал связанных корреляционных функций, а теперь выполняется преобразование Лежандра логарифма с новыми переменными,

Φ α ~ ( Икс , т ) "=" дельта бревно Z [ Дж ~ , Дж ] дельта Дж α ~ ( Икс , т ) Φ α ( Икс , т ) "=" дельта бревно Z [ Дж ~ , Дж ] дельта Дж α ( Икс , т )

В итоге получается производящий функционал для вершинных функций,

Г [ Φ α ~ , Φ α ] "=" бревно Z [ Дж ~ , Дж ] + г г Икс г т α ( Дж α ~ ( Икс , т ) Φ α ~ ( Икс , т ) + Дж α ( Икс , т ) Φ α ( Икс , т ) )

Все это для отображения возмущений корреляционных функций в терминах диаграмм Фейнмана. Связные корреляционные функции будут соответствовать в теории возмущений вкладу связного графа, но, как часто в теории поля, интересуют «реальные» строительные блоки диаграммного разложения, а именно неприводимые одночастичные графы, вклад которых исходит от функционала производные от Г , вершинные функции,

Г { α я } ; { α к } ( Н ~ , Н ) ( { Икс я , т я } ; { Икс к , т к } ) "=" я Н ~ дельта дельта Φ α я ~ ( Икс я , т я ) к Н дельта дельта Φ α к ( Икс к , т к ) Г [ Φ α ~ , Φ α ] | Дж "=" Дж ~ "=" 0

Можно показать, что Г α ; β ( 1 , 1 ) ( Икс , т ; Икс , т ) имеет форму Г ( 1 , 1 ) ( Икс Икс , т т ) дельта α β . В свою очередь, эта форма интересна тем, что после преобразования Фурье мы имеем

Г ( 1 , 1 ) ( д , ю ) "=" г ( д , ю ) 1

Таким образом, с точностью до знака в переменных петлевые диаграммы, дающие вклад в Г ( 1 , 1 ) ( д , ю ) являются не чем иным, как неприводимыми графами собственной энергии одной частицы.

Следовательно, можно вычислить пертурбативное разложение пропагатора, вычислив только вклад этих графиков.


Сама проблема

Чтобы вычислить пертурбативное разложение пропагатора, кажется, вам нужно сначала установить правила Фейнмана для вашей теории, а затем использовать приведенные выше результаты, чтобы утверждать, что вам нужно только вычислить вклады от 1-частичных неприводимых графов.

Но мы также видели явный вид пропагатора в терминах вершинных функций. Давайте посчитаем напрямую Г α ; β ( 1 , 1 ) ( Икс , т ; Икс , т ) затем !

дельта Г [ Φ ~ , Φ ] дельта Φ ~ α ( Икс , т ) "=" Дж ~ α ( Икс , т )

и

дельта 2 Г [ Φ ~ , Φ ] дельта Φ ~ α ( Икс , т ) дельта Φ β ( Икс , т ) "=" дельта Дж ~ α ( Икс , т ) дельта Φ β ( Икс , т ) "=" ?

Первое тождество получить довольно легко. Хотя я вообще не могу вычислить производную тока. Я попытался использовать определение Φ ~ сделать изменение в переменной, но там, кажется, функциональное изменение переменных. Я попробовал обычный метод, т.е. взял многомерный случай и довел число переменных до бесконечности, заменив суммы интегралами, но из этого ничего не вышло.

Есть ли способ продолжить, как это? Является ли это «функциональным изменением переменной» вообще вещью? А если нет, то можно ли вычислить вершинные функции другим способом?


Большое спасибо, что дочитали до сюда. На самом деле я не ожидаю полного ответа (если только г-н Таубер не находится на этом сайте и не увидит мой пост), но любая ссылка либо на соответствующий контент в QFT или SFT, либо на математический текст об этой идее функционального изменения переменной будет высоко оценен.

Пожалуйста, не стесняйтесь спрашивать о какой-либо точности, которую вы хотели бы видеть в этом вопросе, возможно, я был слишком быстр в контексте (хотя он уже слишком длинный).

Примечание: я не мог посещать курсы функционального анализа во время учебы, поэтому у меня были проблемы с поиском чего-либо в статьях по mathSE/math. Тем не менее, я готов нырнуть в случае необходимости.

У вас будет такая же проблема в любом QFT, поэтому, возможно, вам следует взглянуть на более стандартный ф 4 теория. Дело в том, что вы не можете вычислить нужную вам производную, потому что для этого вам нужно знать Г , что вы пытаетесь вычислить... Итак, вам нужно написать Г как расширение цикла и вычислить его производные по порядку.
Спасибо за комментарий ! Вы хотите сказать, что здесь может быть логическая ошибка? На самом деле, у меня есть полное выражение для Г , потому что у меня есть один для Z (это полный формализм MSR, который я несколько пропустил), хотя он все еще находится в функциональной интегральной форме. И я хотел заранее избежать этого расширения, потому что в противном случае я предпочел бы вычислять вклады в пропагатор непосредственно в моем преобразованном Фурье уравнении Ланжевена, скажем. Тем не менее, я посмотрю более подробно на ф 4 .
«хотя он все еще находится в форме функционального интеграла»: это именно то, что я имею в виду под «вам нужно вычислить Г ". У вас есть его неявное непертурбативное определение, которое на самом деле не очень полезно. Другой способ сформулировать это так: если у вас нет полезной правой части уравнения Г "=" . . . (то есть не его формальное определение), вы не можете вычислить Г ( 1 , 1 ) . В противном случае у вас был бы доступ к точному эффективному действию без выполнения каких-либо явных вычислений...
@ Адам Теперь, когда я углубился в это, я определенно понимаю, что ваш последний комментарий - это ответ на мой вопрос. Я приму это как ответ, если вы также укажете, что важно использовать Г ( 1 , 1 ) "=" г 1 с анзацем для Г . Я мог бы сделать это и сам, но я хочу отдать тебе должное за то, что ты был прав до того, как я понял, что ты прав.

Ответы (2)

Здесь на карту поставлено два вопроса.

Во-первых, по самому вопросу:

По определению, Г является преобразованием Лежандра Вт "=" п Z ,

Г [ ф ] "=" Вт [ Дж ] + ф . Дж , дельта Г дельта ф А "=" Дж А ,
где я использую сжатые обозначения Дж А , ф А , Дж . ф "=" Дж А ф А и т.д. с А сбор пространственно-временных координат, индексов и так далее.

Мы также знаем, что дельта Вт дельта Дж А "=" ф А и дельта 2 Вт дельта Дж А дельта Дж Б "=" дельта ф А дельта Дж Б "=" г А Б .

Таким образом, мы находим, что

дельта 2 Г дельта ф А дельта ф Б "=" дельта Дж А дельта ф Б "=" ( дельта ф Б дельта Дж А ) 1 "=" ( г 1 ) А Б .

С точностью до преобразования Фурье это уравнение, данное в ОП. На самом деле это уравнение и то, которое можно получить, взяв больше функциональных производных, просто дают нам связь между связанными корреляционными функциями и вершинной функцией, не более того.

Это подводит нас ко второй проблеме: использование формального определения Г не помогает вычислить ни Г ( 2 ) ни г (если мы не знаем Вт конечно). Таким образом, нам нужен другой способ вычисления Г , а затем используйте это для вычисления г .

Одна из возможностей состоит в том, чтобы вычислить Г как расширение цикла (лучше вычислить Г чем Вт потому что диаграмм меньше (только 1ПИ) и пересуммирование собственных энергий уже выполнено явно).

Существуют и другие приближения, использующие анзац Г и используя некоторое уравнение РГ, чтобы найти коэффициенты (в контексте неравновесной динамики, см., например, Phys. Rev. Lett. 92 , 195703 (2004))

Трюк для вычислений Г ( 1 , 1 ) заключается в вычислении перекрестной производной

дельта 2 Г дельта Φ α ( Икс , т ) дельта Дж β ( Икс , т ) .
Это можно сделать двумя способами. С одной стороны, вы знаете, что результат дельта α β дельта ( Икс Икс ) дельта ( т т ) , потому что это просто производная от Дж α ( Икс , т ) . Но вы также можете вычислить его, используя цепное правило, чтобы выразить производные от Г относительно Дж β ( Икс , т ) в терминах производных от Г относительно Φ α ( Икс , т ) , которая является естественной переменной Г . Вы получаете

г г Икс г т Г α , β ( 1 , 1 ) ( Икс , т ; Икс , т ) г ( Икс Икс , т т ) "=" дельта ( Икс Икс ) дельта ( т т ) дельта α , β
И вы инвертируете свертки в пространстве Фурье, чтобы получить
Г ( 1 , 1 ) ( д , ю ) "=" г 1 ( д , ю ) .

Это делается более подробно в разд. 4.4.2 в книге Таубера.

Большое спасибо за ответ @TomasG, хотя я знал это во время вопроса. Моя проблема заключалась в том, что я не видел, что необходимо иметь анзац для изучаемой теории, а затем использовать упомянутое вами тождество.
Извините, я последовал вашим инструкциям и пропустил часть «контекст», так как знал, о чем вы говорите, и я пропустил эту часть. Я неправильно понял ваш вопрос.